Can x/y approach any real on the real line?

  • Thread starter cragar
  • Start date
  • Tags
    Set
In summary, the conversation discusses the density of a set x/y, where x and y can be any prime, on the real line. It is suggested that the set is dense and a proof is discussed, using the fact that there is a prime between n and 2n where n is a natural number. Another method is proposed, where fractions are constructed and added or subtracted to potentially form prime ratios. However, it is noted that there may be difficulties in ensuring that there are infinitely many n. Finally, a proof is provided using the Prime Number Theorem.
  • #1
cragar
2,552
3
if i have this set x/y where x and y can be any prime. is this dense on the real line.
and we will allow x to be negative so we can cover the negative side of the reals.
It seems like it would be. how would i prove that x/y can approach any real?
 
Mathematics news on Phys.org
  • #2
cragar said:
if i have this set x/y where x and y can be any prime. is this dense on the real line.
and we will allow x to be negative so we can cover the negative side of the reals.
It seems like it would be. how would i prove that x/y can approach any real?

Hmmm. It seems very likely that they are dense. Proving it might not be that easy though: there MIGHT be some exception. The first thing I would try is assume there is such an exception and see whether that leads to a contradiction. Or maybe proving that they are dense on the rationals would be good enough, since the rationals are dense on the reals.
 
  • #3
Well, it seems the only problem you may have is with the fractions 1/k.

Don't you basically get the rationals with the set {x/y: x,y prime} except for {1/k}?

After all, you reduce the rationals {p/q: p,q integer} by eliminating common terms;

you actually eliminate redundancy, but you end up with the same thing: so all you

need, I think , is to deal with density in [0,1/2].
 
  • #4
Bacle2 said:
Don't you basically get the rationals with the set {x/y: x,y prime} except for {1/k}?

Not at all. You actually miss a bunch of rational numbers. For example, you miss all rational numbers of the form p/qn where p,q are distinct primes and 1 < n.
 
  • #5
Right, my bad. I jumped in too quickly.
 
  • #6
Bacle2 said:
Right, my bad. I jumped in too quickly.

Me too. My 'proof' of the result actually fails.
 
  • #7
could we maybe use the fact that there is prime between n and 2n where n is a natural number.
 
  • #8
cragar said:
could we maybe use the fact that there is prime between n and 2n where n is a natural number.

That seems to be the right approach to me. I imagine the best way to go about it would be some sort of quasi-constructive proof, where we take an arbitrary real number and use the above fact (or something like) to show that a prime fraction exists in any neighbourhood of x.
 
  • #9
we could try something similar to how they prove the rationals are dense in the reals.
given any two reals a and b we pick n large enough so that
[itex] \frac{1}{n}<b-a [/itex] then we take the next prime after n and we call it k.
so now we have a prime on the bottom. ok and now we will pick m such that it puts us in between a and b. m is a natural number but not a prime for sure. But we know there is prime between m and 2m. but we may have skipped over b. but we could make the denominator bigger to give us more options for m.
 
Last edited:
  • #10
cragar said:
we could try something similar to how they prove the rationals are dense in the reals.
given any two reals a and b we pick n large enough so that
[itex] \frac{1}{n}<b-a [/itex] then we take the next prime after n and we call it k.
so now we have a prime on the bottom. ok and now we will pick m such that it puts us in between a and b. m is a natural number but not a prime for sure. But we know there is prime between m and 2m. but we may have skipped over b. but we could make the denominator bigger to give us more options for m.

Right, I have been thinking about a method to (see if it is possible ) to do this. But, with

my method, the (numbers in the) fractions become way too large. So, say, we want to

approximate 1/2 within 1/100: so my idea is : consider the set {n/2n} (clearly not

a ratio of primes ), and consider, for fixed n, the ratios:Pr:= {(n-1)/(2n-1), (n+1)/(2n+1),

(n+1)/(2n-1)(n-1)/(2n+1)}, }. If both numerator and denominator are prime, we get

a good approximation:| 1/2 - (n-1)/(2n-1) |= 1/(2n-1) -->0 as n becomes large; similar for other ratios. For

example, with 1/2 itself,

we can consider : 29/59=(30-1)/(60-1), 31/61, 73/37 , 157/79 ,... as approximations.

If we know there are infinitely-many n in the prime-ratio set Pr . Problem is that ,

while there are infinitely-many primes, we cannot guarantee right away ( may need an

additional argument) that the primes beyond a certain point are of this type. I think

this is feasible, but I'm being careful given my previous error.
 
  • #11
Let (x,y) be an open interval in the real numbers with 0 < x < y. The Prime Number Theorem implies that limq→∞[π(qy)-π(qx)] = ∞ and this means that for a sufficiently large prime q, there exists a prime p such that qx < p < qy. Then x < p/q < y as desired.

Unless I messed up on the computation of limq→∞[π(qy)-π(qx)], then this proof should work fine.
 
  • #12
What do you think about my idea:
What do you think about my argument?

My idea is , e.g., for x=1/2, to construct the set :

{1/2,2/4,...,2n/4n,...}

Then, to each term in the sequence , we add/subtract 1 to each numerator and

denominator, so , e.g:

1/2 --> 2/3, 2/1 , 0/1, 0/3

2/4 --> 3/5, 3/3, 1/3, 1/5 ,

etc.

Each of these terms is a potential ratio of primes, and a good approximation to 1/2,

with the caveat that primes of this type must be infinitely-many.
 
  • #13
I thought about a similar approach to yours yesterday, but I came across too many difficulties trying to make everything work. The problem is that for a fixed rational number p/q it is difficult to ensure that there are infinitely many n for which both np-1 and nq-1 are prime. Using something like Dirichlet's Theorem you should be able to show that there are infinitely many n with np-1 prime and infinitely many n with nq-1 prime, but this (unfortunately) says nothing about n for which both of them are prime. There might be a clever way around this or there might be some theorem which guarantees the existence of infinitely many such n, but with my very limited knowledge I obviously know of neither.
 
  • #14
Thanks for the input, Jgens.
 

1. What is a dense set?

A dense set is a subset of a topological space in which every point in the space lies either in the set itself or in its closure. This means that given any point in the space, it is either part of the set or arbitrarily close to it.

2. How is a dense set different from a compact set?

A dense set is defined by the property that every point in the space is either in the set or in its closure. A compact set, on the other hand, is defined by the property that it is both closed and bounded.

3. What is the importance of dense sets in mathematics?

Dense sets are important in various branches of mathematics, such as topology, analysis, and measure theory. They provide a way to approximate points in a space and can be used to prove the existence of certain objects or solutions to problems.

4. How can one prove that a set is dense?

To prove that a set is dense, one must show that every point in the space is either in the set or in its closure. This can be done by using the definition of a dense set and showing that for any point in the space, there exists a sequence of points in the set that converges to that point.

5. Are there different types of dense sets?

Yes, there are different types of dense sets depending on the topological space in which they are defined. Some examples include countable dense sets, dense-in-itself sets, and nowhere dense sets. Each type has its own set of properties and can be used in different mathematical contexts.

Similar threads

Replies
2
Views
246
  • General Math
Replies
7
Views
1K
Replies
15
Views
1K
  • General Math
Replies
5
Views
842
Replies
4
Views
976
  • General Math
Replies
21
Views
4K
Replies
7
Views
886
Replies
4
Views
852
  • General Math
Replies
6
Views
1K
Replies
3
Views
703
Back
Top